LSAT Explanation PT 42, S2, Q1: Carl is clearly an incompetent detective.
LSAT Question Stem
Which one of the following, if true, most seriously weakens the argument above?
Logical Reasoning Question Type
This is a Weaken question.
Correct Answer
The correct answer to this question is A.
LSAT Question Complete Explanation
The question type for this problem is Weaken, and our goal is to find an answer choice that undermines the conclusion of the argument without attacking the premises.
Argument Summary:
Premise: Carl has solved a smaller percentage of cases assigned to him in the last 3 years (1 out of 25) than any other detective on the police force.
Conclusion: Carl is an incompetent detective.
To better understand this argument, let's use a simple example. Imagine a classroom where a student named Sally has the lowest test scores. The argument would be that since Sally has the lowest test scores, she must be the least intelligent student in the class. However, this conclusion might not be accurate if there are other factors at play, such as Sally having a more challenging course load than her peers.
Evaluate Question: Are there any factors that could explain Carl's low success rate other than his incompetence?
Now, let's analyze each answer choice:
a) This answer choice provides a relevant piece of information that could explain Carl's low success rate: he is assigned the most difficult cases, which are likely to have a lower success rate. This weakens the argument by suggesting that Carl's low success rate might be due to the difficulty of his cases rather than his incompetence. Therefore, this is the correct answer.
b) This answer choice is irrelevant. Carl's past performance as a neighborhood police officer does not directly address his current success rate as a detective. It does not provide any information that would weaken the argument.
c) This answer choice actually strengthens the argument by showing that Carl has access to extensive resources to help him solve crimes. If he still has a low success rate despite these resources, it might suggest that he is indeed incompetent.
d) This answer choice also strengthens the argument by providing evidence of Carl's poor performance in another city. This further supports the conclusion that Carl is an incompetent detective.
e) This answer choice goes beyond the scope of the argument by discussing the hiring and promotion of other officers. It does not provide any information about Carl's performance or any factors that could explain his low success rate.
In conclusion, the correct answer choice is (a) because it provides a plausible explanation for Carl's low success rate other than his incompetence, thus weakening the argument.
